Why does $int_0^infty cos x^2 dx$ converge? The Next CEO of Stack Overflow$iint_D cos left(...

How to avoid supervisors with prejudiced views?

Are there any unintended negative consequences to allowing PCs to gain multiple levels at once in a short milestone-XP game?

Which tube will fit a -(700 x 25c) wheel?

To not tell, not take, and not want

Is it professional to write unrelated content in an almost-empty email?

Rotate a column

How does the Z80 determine which peripheral sent an interrupt?

How to invert MapIndexed on a ragged structure? How to construct a tree from rules?

Is it my responsibility to learn a new technology in my own time my employer wants to implement?

Inappropriate reference requests from Journal reviewers

Do I need to enable Dev Hub in my PROD Org?

What exact does MIB represent in SNMP? How is it different from OID?

How do I avoid eval and parse?

Which kind of appliances can one connect to electric sockets located in a airplane's toilet?

Sending manuscript to multiple publishers

Can I equip Skullclamp on a creature I am sacrificing?

What's the best way to handle refactoring a big file?

What is "(CFMCC)" on an ILS approach chart?

Should I tutor a student who I know has cheated on their homework?

Why don't programming languages automatically manage the synchronous/asynchronous problem?

How to count occurrences of text in a file?

What benefits would be gained by using human laborers instead of drones in deep sea mining?

Would a completely good Muggle be able to use a wand?

How do I reset passwords on multiple websites easily?



Why does $int_0^infty cos x^2 dx$ converge?



The Next CEO of Stack Overflow$iint_D cos left( frac{x-y}{x+y} right),dA$Comparison test for series $sum_{n=1}^{infty}frac{n}{n^3 - 2n + 1}$Simple question about Integral from 0 to f(x) instead of xConvergent subsequences of $x_n = sin n$ and $y_n = cos n$…How to integrate $e^{sin x}(x cos x - tan x sec x)$Does the sum $sum_{n=1}^{infty}{a_nb_n}$ converge(fourier series coefficients)?Discuss the convergence of $int_0^infty x sin e^x , dx$Find $int_0^infty frac{sin(4x)}{x}$Limit of $lim_{t to infty} frac{ int_0^infty cos(x t) e^{-x^k}dx}{int_0^infty cos(x t) e^{-x^p}dx}$Prove that $int_0^{infty} frac{sin x}{x^p}, dx$ converges for $0<p<2$












0












$begingroup$


I'm a B.C. student. I took a test and a question was:




does $int_0^infty cos x^2 dx$ converge?




  • Yes

  • No




I undoubtedly answered No because this is going to be somehow periodically, not exactly periodically but after any given number, there is a greater number $s$ that $int_0^s cos x^2 dx = 0$ and a little bit after $s$ the value is not zero. because it is $cos$ you know!



But after the test, I found out that the right answer is Yes and I was wrong.



Now I'm thinking for hours and I really really can't understand it. please help.










share|cite|improve this question









$endgroup$








  • 1




    $begingroup$
    related: en.wikipedia.org/wiki/Fresnel_integral#Properties
    $endgroup$
    – Math-fun
    Mar 16 at 18:56
















0












$begingroup$


I'm a B.C. student. I took a test and a question was:




does $int_0^infty cos x^2 dx$ converge?




  • Yes

  • No




I undoubtedly answered No because this is going to be somehow periodically, not exactly periodically but after any given number, there is a greater number $s$ that $int_0^s cos x^2 dx = 0$ and a little bit after $s$ the value is not zero. because it is $cos$ you know!



But after the test, I found out that the right answer is Yes and I was wrong.



Now I'm thinking for hours and I really really can't understand it. please help.










share|cite|improve this question









$endgroup$








  • 1




    $begingroup$
    related: en.wikipedia.org/wiki/Fresnel_integral#Properties
    $endgroup$
    – Math-fun
    Mar 16 at 18:56














0












0








0


1



$begingroup$


I'm a B.C. student. I took a test and a question was:




does $int_0^infty cos x^2 dx$ converge?




  • Yes

  • No




I undoubtedly answered No because this is going to be somehow periodically, not exactly periodically but after any given number, there is a greater number $s$ that $int_0^s cos x^2 dx = 0$ and a little bit after $s$ the value is not zero. because it is $cos$ you know!



But after the test, I found out that the right answer is Yes and I was wrong.



Now I'm thinking for hours and I really really can't understand it. please help.










share|cite|improve this question









$endgroup$




I'm a B.C. student. I took a test and a question was:




does $int_0^infty cos x^2 dx$ converge?




  • Yes

  • No




I undoubtedly answered No because this is going to be somehow periodically, not exactly periodically but after any given number, there is a greater number $s$ that $int_0^s cos x^2 dx = 0$ and a little bit after $s$ the value is not zero. because it is $cos$ you know!



But after the test, I found out that the right answer is Yes and I was wrong.



Now I'm thinking for hours and I really really can't understand it. please help.







integration convergence






share|cite|improve this question













share|cite|improve this question











share|cite|improve this question




share|cite|improve this question










asked Mar 16 at 18:33









Peyman mohseni kiasariPeyman mohseni kiasari

13711




13711








  • 1




    $begingroup$
    related: en.wikipedia.org/wiki/Fresnel_integral#Properties
    $endgroup$
    – Math-fun
    Mar 16 at 18:56














  • 1




    $begingroup$
    related: en.wikipedia.org/wiki/Fresnel_integral#Properties
    $endgroup$
    – Math-fun
    Mar 16 at 18:56








1




1




$begingroup$
related: en.wikipedia.org/wiki/Fresnel_integral#Properties
$endgroup$
– Math-fun
Mar 16 at 18:56




$begingroup$
related: en.wikipedia.org/wiki/Fresnel_integral#Properties
$endgroup$
– Math-fun
Mar 16 at 18:56










3 Answers
3






active

oldest

votes


















7












$begingroup$

To use some elementary methods to prove this integral converges, consider that $$int_0^infty cos(x^2),dx=int_0^1cos(x^2),dx+int_1^inftycos(x^2),dx.$$ The first integral obviously converges since $|cos(x^2)|leq1$. For the second integral, we have $$int_1^inftyfrac{1}{x},xcos(x^2),dx=frac{sin(x^2)}{2x}bigg|_{x=1}^{x=infty}+int_1^inftyfrac{sin(x^2)}{2x^2},dx.$$ This equality follows from integration by parts; the first term above is obviously convergent, while the integral can be bounded above, since $$left|frac{sin(x^2)}{2x^2}right|leqfrac{1}{2x^2},$$and $int x^{-2},dx$ converges as can be shown directly using the power rule for integration.






share|cite|improve this answer









$endgroup$













  • $begingroup$
    and finally, it is also good to mention that it converges to $sqrt{frac{pi}{8}}$ thanks to en.wikipedia.org/wiki/Fresnel_integral
    $endgroup$
    – Peyman mohseni kiasari
    Mar 16 at 19:00






  • 1




    $begingroup$
    @Peymanmohsenikiasari: Indeed! To determine the exact value the integral converges to resorts to using complex analysis (contour integration). I thought this might be a bit above a BC calculus class, so I didn't use it and instead chose to use elementary methods. There might be elementary methods to prove the exact value, but I'm not aware of any off the top of my head.
    $endgroup$
    – Clayton
    Mar 16 at 19:03










  • $begingroup$
    your answer was great and so helpful, thank you. I just mention that for "more information" for whoever visit this page in the future.
    $endgroup$
    – Peyman mohseni kiasari
    Mar 16 at 19:07



















1












$begingroup$

Don't be deceived; although $cos x^2$ oscillates, it does at a frequency which increases as $x,$ which makes the argument go to $+infty$ as $x$ becomes arbitrarily large. Therefore, although it oscillates, it does so in such a way that it adds and subtracts sufficiently smaller and smaller areas, and does so faster than $x$ is running away, in order to converge.





I have given a heuristic explanation in the spirit of OP






share|cite|improve this answer











$endgroup$









  • 1




    $begingroup$
    Surely the frequency scales as $x$, not $x^2$, since it takes a $O(1/x)$ change in a large $x$ to increase $x^2$ by $2pi$.
    $endgroup$
    – J.G.
    Mar 16 at 18:51










  • $begingroup$
    thanks, nice explanation! what a tricky and beautiful convergence problem it was.
    $endgroup$
    – Peyman mohseni kiasari
    Mar 16 at 18:52










  • $begingroup$
    @J.G. Sure, that's true. Thanks, I got carried away by the extremely loose way I was talking.
    $endgroup$
    – Allawonder
    Mar 16 at 18:58



















0












$begingroup$

The usual idea of a function decaying sufficiently fast for convergence of the improper integral does not apply to sign-changing functions. The reason for the convergence in this case is the highly oscillatory character of the function $cos x^2$ at infinity. The positive and negative "areas" cancel each other and the integral converges.






share|cite|improve this answer









$endgroup$













  • $begingroup$
    I don't think that we can say "they cancel each other" that easily. why in cos x^2 "the areas cancel each other" but not in cos ln x? I think this needs better math proof.
    $endgroup$
    – Peyman mohseni kiasari
    Mar 16 at 18:46










  • $begingroup$
    I agree. That was just a rough explanation to make convergence plausible. To justify the claim, one should apply Dirichlet's or Abel's criteria, or a nice elementary argument like Clayton's.
    $endgroup$
    – GReyes
    Mar 16 at 20:13














Your Answer





StackExchange.ifUsing("editor", function () {
return StackExchange.using("mathjaxEditing", function () {
StackExchange.MarkdownEditor.creationCallbacks.add(function (editor, postfix) {
StackExchange.mathjaxEditing.prepareWmdForMathJax(editor, postfix, [["$", "$"], ["\\(","\\)"]]);
});
});
}, "mathjax-editing");

StackExchange.ready(function() {
var channelOptions = {
tags: "".split(" "),
id: "69"
};
initTagRenderer("".split(" "), "".split(" "), channelOptions);

StackExchange.using("externalEditor", function() {
// Have to fire editor after snippets, if snippets enabled
if (StackExchange.settings.snippets.snippetsEnabled) {
StackExchange.using("snippets", function() {
createEditor();
});
}
else {
createEditor();
}
});

function createEditor() {
StackExchange.prepareEditor({
heartbeatType: 'answer',
autoActivateHeartbeat: false,
convertImagesToLinks: true,
noModals: true,
showLowRepImageUploadWarning: true,
reputationToPostImages: 10,
bindNavPrevention: true,
postfix: "",
imageUploader: {
brandingHtml: "Powered by u003ca class="icon-imgur-white" href="https://imgur.com/"u003eu003c/au003e",
contentPolicyHtml: "User contributions licensed under u003ca href="https://creativecommons.org/licenses/by-sa/3.0/"u003ecc by-sa 3.0 with attribution requiredu003c/au003e u003ca href="https://stackoverflow.com/legal/content-policy"u003e(content policy)u003c/au003e",
allowUrls: true
},
noCode: true, onDemand: true,
discardSelector: ".discard-answer"
,immediatelyShowMarkdownHelp:true
});


}
});














draft saved

draft discarded


















StackExchange.ready(
function () {
StackExchange.openid.initPostLogin('.new-post-login', 'https%3a%2f%2fmath.stackexchange.com%2fquestions%2f3150662%2fwhy-does-int-0-infty-cos-x2-dx-converge%23new-answer', 'question_page');
}
);

Post as a guest















Required, but never shown

























3 Answers
3






active

oldest

votes








3 Answers
3






active

oldest

votes









active

oldest

votes






active

oldest

votes









7












$begingroup$

To use some elementary methods to prove this integral converges, consider that $$int_0^infty cos(x^2),dx=int_0^1cos(x^2),dx+int_1^inftycos(x^2),dx.$$ The first integral obviously converges since $|cos(x^2)|leq1$. For the second integral, we have $$int_1^inftyfrac{1}{x},xcos(x^2),dx=frac{sin(x^2)}{2x}bigg|_{x=1}^{x=infty}+int_1^inftyfrac{sin(x^2)}{2x^2},dx.$$ This equality follows from integration by parts; the first term above is obviously convergent, while the integral can be bounded above, since $$left|frac{sin(x^2)}{2x^2}right|leqfrac{1}{2x^2},$$and $int x^{-2},dx$ converges as can be shown directly using the power rule for integration.






share|cite|improve this answer









$endgroup$













  • $begingroup$
    and finally, it is also good to mention that it converges to $sqrt{frac{pi}{8}}$ thanks to en.wikipedia.org/wiki/Fresnel_integral
    $endgroup$
    – Peyman mohseni kiasari
    Mar 16 at 19:00






  • 1




    $begingroup$
    @Peymanmohsenikiasari: Indeed! To determine the exact value the integral converges to resorts to using complex analysis (contour integration). I thought this might be a bit above a BC calculus class, so I didn't use it and instead chose to use elementary methods. There might be elementary methods to prove the exact value, but I'm not aware of any off the top of my head.
    $endgroup$
    – Clayton
    Mar 16 at 19:03










  • $begingroup$
    your answer was great and so helpful, thank you. I just mention that for "more information" for whoever visit this page in the future.
    $endgroup$
    – Peyman mohseni kiasari
    Mar 16 at 19:07
















7












$begingroup$

To use some elementary methods to prove this integral converges, consider that $$int_0^infty cos(x^2),dx=int_0^1cos(x^2),dx+int_1^inftycos(x^2),dx.$$ The first integral obviously converges since $|cos(x^2)|leq1$. For the second integral, we have $$int_1^inftyfrac{1}{x},xcos(x^2),dx=frac{sin(x^2)}{2x}bigg|_{x=1}^{x=infty}+int_1^inftyfrac{sin(x^2)}{2x^2},dx.$$ This equality follows from integration by parts; the first term above is obviously convergent, while the integral can be bounded above, since $$left|frac{sin(x^2)}{2x^2}right|leqfrac{1}{2x^2},$$and $int x^{-2},dx$ converges as can be shown directly using the power rule for integration.






share|cite|improve this answer









$endgroup$













  • $begingroup$
    and finally, it is also good to mention that it converges to $sqrt{frac{pi}{8}}$ thanks to en.wikipedia.org/wiki/Fresnel_integral
    $endgroup$
    – Peyman mohseni kiasari
    Mar 16 at 19:00






  • 1




    $begingroup$
    @Peymanmohsenikiasari: Indeed! To determine the exact value the integral converges to resorts to using complex analysis (contour integration). I thought this might be a bit above a BC calculus class, so I didn't use it and instead chose to use elementary methods. There might be elementary methods to prove the exact value, but I'm not aware of any off the top of my head.
    $endgroup$
    – Clayton
    Mar 16 at 19:03










  • $begingroup$
    your answer was great and so helpful, thank you. I just mention that for "more information" for whoever visit this page in the future.
    $endgroup$
    – Peyman mohseni kiasari
    Mar 16 at 19:07














7












7








7





$begingroup$

To use some elementary methods to prove this integral converges, consider that $$int_0^infty cos(x^2),dx=int_0^1cos(x^2),dx+int_1^inftycos(x^2),dx.$$ The first integral obviously converges since $|cos(x^2)|leq1$. For the second integral, we have $$int_1^inftyfrac{1}{x},xcos(x^2),dx=frac{sin(x^2)}{2x}bigg|_{x=1}^{x=infty}+int_1^inftyfrac{sin(x^2)}{2x^2},dx.$$ This equality follows from integration by parts; the first term above is obviously convergent, while the integral can be bounded above, since $$left|frac{sin(x^2)}{2x^2}right|leqfrac{1}{2x^2},$$and $int x^{-2},dx$ converges as can be shown directly using the power rule for integration.






share|cite|improve this answer









$endgroup$



To use some elementary methods to prove this integral converges, consider that $$int_0^infty cos(x^2),dx=int_0^1cos(x^2),dx+int_1^inftycos(x^2),dx.$$ The first integral obviously converges since $|cos(x^2)|leq1$. For the second integral, we have $$int_1^inftyfrac{1}{x},xcos(x^2),dx=frac{sin(x^2)}{2x}bigg|_{x=1}^{x=infty}+int_1^inftyfrac{sin(x^2)}{2x^2},dx.$$ This equality follows from integration by parts; the first term above is obviously convergent, while the integral can be bounded above, since $$left|frac{sin(x^2)}{2x^2}right|leqfrac{1}{2x^2},$$and $int x^{-2},dx$ converges as can be shown directly using the power rule for integration.







share|cite|improve this answer












share|cite|improve this answer



share|cite|improve this answer










answered Mar 16 at 18:46









ClaytonClayton

19.5k33288




19.5k33288












  • $begingroup$
    and finally, it is also good to mention that it converges to $sqrt{frac{pi}{8}}$ thanks to en.wikipedia.org/wiki/Fresnel_integral
    $endgroup$
    – Peyman mohseni kiasari
    Mar 16 at 19:00






  • 1




    $begingroup$
    @Peymanmohsenikiasari: Indeed! To determine the exact value the integral converges to resorts to using complex analysis (contour integration). I thought this might be a bit above a BC calculus class, so I didn't use it and instead chose to use elementary methods. There might be elementary methods to prove the exact value, but I'm not aware of any off the top of my head.
    $endgroup$
    – Clayton
    Mar 16 at 19:03










  • $begingroup$
    your answer was great and so helpful, thank you. I just mention that for "more information" for whoever visit this page in the future.
    $endgroup$
    – Peyman mohseni kiasari
    Mar 16 at 19:07


















  • $begingroup$
    and finally, it is also good to mention that it converges to $sqrt{frac{pi}{8}}$ thanks to en.wikipedia.org/wiki/Fresnel_integral
    $endgroup$
    – Peyman mohseni kiasari
    Mar 16 at 19:00






  • 1




    $begingroup$
    @Peymanmohsenikiasari: Indeed! To determine the exact value the integral converges to resorts to using complex analysis (contour integration). I thought this might be a bit above a BC calculus class, so I didn't use it and instead chose to use elementary methods. There might be elementary methods to prove the exact value, but I'm not aware of any off the top of my head.
    $endgroup$
    – Clayton
    Mar 16 at 19:03










  • $begingroup$
    your answer was great and so helpful, thank you. I just mention that for "more information" for whoever visit this page in the future.
    $endgroup$
    – Peyman mohseni kiasari
    Mar 16 at 19:07
















$begingroup$
and finally, it is also good to mention that it converges to $sqrt{frac{pi}{8}}$ thanks to en.wikipedia.org/wiki/Fresnel_integral
$endgroup$
– Peyman mohseni kiasari
Mar 16 at 19:00




$begingroup$
and finally, it is also good to mention that it converges to $sqrt{frac{pi}{8}}$ thanks to en.wikipedia.org/wiki/Fresnel_integral
$endgroup$
– Peyman mohseni kiasari
Mar 16 at 19:00




1




1




$begingroup$
@Peymanmohsenikiasari: Indeed! To determine the exact value the integral converges to resorts to using complex analysis (contour integration). I thought this might be a bit above a BC calculus class, so I didn't use it and instead chose to use elementary methods. There might be elementary methods to prove the exact value, but I'm not aware of any off the top of my head.
$endgroup$
– Clayton
Mar 16 at 19:03




$begingroup$
@Peymanmohsenikiasari: Indeed! To determine the exact value the integral converges to resorts to using complex analysis (contour integration). I thought this might be a bit above a BC calculus class, so I didn't use it and instead chose to use elementary methods. There might be elementary methods to prove the exact value, but I'm not aware of any off the top of my head.
$endgroup$
– Clayton
Mar 16 at 19:03












$begingroup$
your answer was great and so helpful, thank you. I just mention that for "more information" for whoever visit this page in the future.
$endgroup$
– Peyman mohseni kiasari
Mar 16 at 19:07




$begingroup$
your answer was great and so helpful, thank you. I just mention that for "more information" for whoever visit this page in the future.
$endgroup$
– Peyman mohseni kiasari
Mar 16 at 19:07











1












$begingroup$

Don't be deceived; although $cos x^2$ oscillates, it does at a frequency which increases as $x,$ which makes the argument go to $+infty$ as $x$ becomes arbitrarily large. Therefore, although it oscillates, it does so in such a way that it adds and subtracts sufficiently smaller and smaller areas, and does so faster than $x$ is running away, in order to converge.





I have given a heuristic explanation in the spirit of OP






share|cite|improve this answer











$endgroup$









  • 1




    $begingroup$
    Surely the frequency scales as $x$, not $x^2$, since it takes a $O(1/x)$ change in a large $x$ to increase $x^2$ by $2pi$.
    $endgroup$
    – J.G.
    Mar 16 at 18:51










  • $begingroup$
    thanks, nice explanation! what a tricky and beautiful convergence problem it was.
    $endgroup$
    – Peyman mohseni kiasari
    Mar 16 at 18:52










  • $begingroup$
    @J.G. Sure, that's true. Thanks, I got carried away by the extremely loose way I was talking.
    $endgroup$
    – Allawonder
    Mar 16 at 18:58
















1












$begingroup$

Don't be deceived; although $cos x^2$ oscillates, it does at a frequency which increases as $x,$ which makes the argument go to $+infty$ as $x$ becomes arbitrarily large. Therefore, although it oscillates, it does so in such a way that it adds and subtracts sufficiently smaller and smaller areas, and does so faster than $x$ is running away, in order to converge.





I have given a heuristic explanation in the spirit of OP






share|cite|improve this answer











$endgroup$









  • 1




    $begingroup$
    Surely the frequency scales as $x$, not $x^2$, since it takes a $O(1/x)$ change in a large $x$ to increase $x^2$ by $2pi$.
    $endgroup$
    – J.G.
    Mar 16 at 18:51










  • $begingroup$
    thanks, nice explanation! what a tricky and beautiful convergence problem it was.
    $endgroup$
    – Peyman mohseni kiasari
    Mar 16 at 18:52










  • $begingroup$
    @J.G. Sure, that's true. Thanks, I got carried away by the extremely loose way I was talking.
    $endgroup$
    – Allawonder
    Mar 16 at 18:58














1












1








1





$begingroup$

Don't be deceived; although $cos x^2$ oscillates, it does at a frequency which increases as $x,$ which makes the argument go to $+infty$ as $x$ becomes arbitrarily large. Therefore, although it oscillates, it does so in such a way that it adds and subtracts sufficiently smaller and smaller areas, and does so faster than $x$ is running away, in order to converge.





I have given a heuristic explanation in the spirit of OP






share|cite|improve this answer











$endgroup$



Don't be deceived; although $cos x^2$ oscillates, it does at a frequency which increases as $x,$ which makes the argument go to $+infty$ as $x$ becomes arbitrarily large. Therefore, although it oscillates, it does so in such a way that it adds and subtracts sufficiently smaller and smaller areas, and does so faster than $x$ is running away, in order to converge.





I have given a heuristic explanation in the spirit of OP







share|cite|improve this answer














share|cite|improve this answer



share|cite|improve this answer








edited Mar 16 at 19:00

























answered Mar 16 at 18:48









AllawonderAllawonder

1




1








  • 1




    $begingroup$
    Surely the frequency scales as $x$, not $x^2$, since it takes a $O(1/x)$ change in a large $x$ to increase $x^2$ by $2pi$.
    $endgroup$
    – J.G.
    Mar 16 at 18:51










  • $begingroup$
    thanks, nice explanation! what a tricky and beautiful convergence problem it was.
    $endgroup$
    – Peyman mohseni kiasari
    Mar 16 at 18:52










  • $begingroup$
    @J.G. Sure, that's true. Thanks, I got carried away by the extremely loose way I was talking.
    $endgroup$
    – Allawonder
    Mar 16 at 18:58














  • 1




    $begingroup$
    Surely the frequency scales as $x$, not $x^2$, since it takes a $O(1/x)$ change in a large $x$ to increase $x^2$ by $2pi$.
    $endgroup$
    – J.G.
    Mar 16 at 18:51










  • $begingroup$
    thanks, nice explanation! what a tricky and beautiful convergence problem it was.
    $endgroup$
    – Peyman mohseni kiasari
    Mar 16 at 18:52










  • $begingroup$
    @J.G. Sure, that's true. Thanks, I got carried away by the extremely loose way I was talking.
    $endgroup$
    – Allawonder
    Mar 16 at 18:58








1




1




$begingroup$
Surely the frequency scales as $x$, not $x^2$, since it takes a $O(1/x)$ change in a large $x$ to increase $x^2$ by $2pi$.
$endgroup$
– J.G.
Mar 16 at 18:51




$begingroup$
Surely the frequency scales as $x$, not $x^2$, since it takes a $O(1/x)$ change in a large $x$ to increase $x^2$ by $2pi$.
$endgroup$
– J.G.
Mar 16 at 18:51












$begingroup$
thanks, nice explanation! what a tricky and beautiful convergence problem it was.
$endgroup$
– Peyman mohseni kiasari
Mar 16 at 18:52




$begingroup$
thanks, nice explanation! what a tricky and beautiful convergence problem it was.
$endgroup$
– Peyman mohseni kiasari
Mar 16 at 18:52












$begingroup$
@J.G. Sure, that's true. Thanks, I got carried away by the extremely loose way I was talking.
$endgroup$
– Allawonder
Mar 16 at 18:58




$begingroup$
@J.G. Sure, that's true. Thanks, I got carried away by the extremely loose way I was talking.
$endgroup$
– Allawonder
Mar 16 at 18:58











0












$begingroup$

The usual idea of a function decaying sufficiently fast for convergence of the improper integral does not apply to sign-changing functions. The reason for the convergence in this case is the highly oscillatory character of the function $cos x^2$ at infinity. The positive and negative "areas" cancel each other and the integral converges.






share|cite|improve this answer









$endgroup$













  • $begingroup$
    I don't think that we can say "they cancel each other" that easily. why in cos x^2 "the areas cancel each other" but not in cos ln x? I think this needs better math proof.
    $endgroup$
    – Peyman mohseni kiasari
    Mar 16 at 18:46










  • $begingroup$
    I agree. That was just a rough explanation to make convergence plausible. To justify the claim, one should apply Dirichlet's or Abel's criteria, or a nice elementary argument like Clayton's.
    $endgroup$
    – GReyes
    Mar 16 at 20:13


















0












$begingroup$

The usual idea of a function decaying sufficiently fast for convergence of the improper integral does not apply to sign-changing functions. The reason for the convergence in this case is the highly oscillatory character of the function $cos x^2$ at infinity. The positive and negative "areas" cancel each other and the integral converges.






share|cite|improve this answer









$endgroup$













  • $begingroup$
    I don't think that we can say "they cancel each other" that easily. why in cos x^2 "the areas cancel each other" but not in cos ln x? I think this needs better math proof.
    $endgroup$
    – Peyman mohseni kiasari
    Mar 16 at 18:46










  • $begingroup$
    I agree. That was just a rough explanation to make convergence plausible. To justify the claim, one should apply Dirichlet's or Abel's criteria, or a nice elementary argument like Clayton's.
    $endgroup$
    – GReyes
    Mar 16 at 20:13
















0












0








0





$begingroup$

The usual idea of a function decaying sufficiently fast for convergence of the improper integral does not apply to sign-changing functions. The reason for the convergence in this case is the highly oscillatory character of the function $cos x^2$ at infinity. The positive and negative "areas" cancel each other and the integral converges.






share|cite|improve this answer









$endgroup$



The usual idea of a function decaying sufficiently fast for convergence of the improper integral does not apply to sign-changing functions. The reason for the convergence in this case is the highly oscillatory character of the function $cos x^2$ at infinity. The positive and negative "areas" cancel each other and the integral converges.







share|cite|improve this answer












share|cite|improve this answer



share|cite|improve this answer










answered Mar 16 at 18:39









GReyesGReyes

2,32315




2,32315












  • $begingroup$
    I don't think that we can say "they cancel each other" that easily. why in cos x^2 "the areas cancel each other" but not in cos ln x? I think this needs better math proof.
    $endgroup$
    – Peyman mohseni kiasari
    Mar 16 at 18:46










  • $begingroup$
    I agree. That was just a rough explanation to make convergence plausible. To justify the claim, one should apply Dirichlet's or Abel's criteria, or a nice elementary argument like Clayton's.
    $endgroup$
    – GReyes
    Mar 16 at 20:13




















  • $begingroup$
    I don't think that we can say "they cancel each other" that easily. why in cos x^2 "the areas cancel each other" but not in cos ln x? I think this needs better math proof.
    $endgroup$
    – Peyman mohseni kiasari
    Mar 16 at 18:46










  • $begingroup$
    I agree. That was just a rough explanation to make convergence plausible. To justify the claim, one should apply Dirichlet's or Abel's criteria, or a nice elementary argument like Clayton's.
    $endgroup$
    – GReyes
    Mar 16 at 20:13


















$begingroup$
I don't think that we can say "they cancel each other" that easily. why in cos x^2 "the areas cancel each other" but not in cos ln x? I think this needs better math proof.
$endgroup$
– Peyman mohseni kiasari
Mar 16 at 18:46




$begingroup$
I don't think that we can say "they cancel each other" that easily. why in cos x^2 "the areas cancel each other" but not in cos ln x? I think this needs better math proof.
$endgroup$
– Peyman mohseni kiasari
Mar 16 at 18:46












$begingroup$
I agree. That was just a rough explanation to make convergence plausible. To justify the claim, one should apply Dirichlet's or Abel's criteria, or a nice elementary argument like Clayton's.
$endgroup$
– GReyes
Mar 16 at 20:13






$begingroup$
I agree. That was just a rough explanation to make convergence plausible. To justify the claim, one should apply Dirichlet's or Abel's criteria, or a nice elementary argument like Clayton's.
$endgroup$
– GReyes
Mar 16 at 20:13




















draft saved

draft discarded




















































Thanks for contributing an answer to Mathematics Stack Exchange!


  • Please be sure to answer the question. Provide details and share your research!

But avoid



  • Asking for help, clarification, or responding to other answers.

  • Making statements based on opinion; back them up with references or personal experience.


Use MathJax to format equations. MathJax reference.


To learn more, see our tips on writing great answers.




draft saved


draft discarded














StackExchange.ready(
function () {
StackExchange.openid.initPostLogin('.new-post-login', 'https%3a%2f%2fmath.stackexchange.com%2fquestions%2f3150662%2fwhy-does-int-0-infty-cos-x2-dx-converge%23new-answer', 'question_page');
}
);

Post as a guest















Required, but never shown





















































Required, but never shown














Required, but never shown












Required, but never shown







Required, but never shown

































Required, but never shown














Required, but never shown












Required, but never shown







Required, but never shown







Popular posts from this blog

Magento 2 - Add success message with knockout Planned maintenance scheduled April 23, 2019 at 23:30 UTC (7:30pm US/Eastern) Announcing the arrival of Valued Associate #679: Cesar Manara Unicorn Meta Zoo #1: Why another podcast?Success / Error message on ajax request$.widget is not a function when loading a homepage after add custom jQuery on custom themeHow can bind jQuery to current document in Magento 2 When template load by ajaxRedirect page using plugin in Magento 2Magento 2 - Update quantity and totals of cart page without page reload?Magento 2: Quote data not loaded on knockout checkoutMagento 2 : I need to change add to cart success message after adding product into cart through pluginMagento 2.2.5 How to add additional products to cart from new checkout step?Magento 2 Add error/success message with knockoutCan't validate Post Code on checkout page

Fil:Tokke komm.svg

Where did Arya get these scars? Unicorn Meta Zoo #1: Why another podcast? Announcing the arrival of Valued Associate #679: Cesar Manara Favourite questions and answers from the 1st quarter of 2019Why did Arya refuse to end it?Has the pronunciation of Arya Stark's name changed?Has Arya forgiven people?Why did Arya Stark lose her vision?Why can Arya still use the faces?Has the Narrow Sea become narrower?Does Arya Stark know how to make poisons outside of the House of Black and White?Why did Nymeria leave Arya?Why did Arya not kill the Lannister soldiers she encountered in the Riverlands?What is the current canonical age of Sansa, Bran and Arya Stark?